LSAT and Law School Admissions Forum

Get expert LSAT preparation and law school admissions advice from PowerScore Test Preparation.

User avatar
 Dave Killoran
PowerScore Staff
  • PowerScore Staff
  • Posts: 5852
  • Joined: Mar 25, 2011
|
#27192
Complete Question Explanation
(The complete setup for this game can be found here: lsat/viewtopic.php?t=11429)

The correct answer choice is (A)

Irrespective of the Local conditions, answer choices (B), (D), and (E) can be eliminated by the PM and day 1 Not Laws. With the answers narrowed to (A) and (C), we can consider the Local condition in the question stem. The stem produces the following mini-diagram next to the question:
Dec 94_M12_game#3_L7_explanations_game#1_#13_diagram_1.png

Note that in doing the question on the test, we would not take the time to write out AM, PM, 1, 2, and 3.

This setup automatically rules out P’s being scheduled on day 1, since if P is scheduled for day 1, then both F and H have to be scheduled for day 2. Since F is already scheduled for day 3, answer choice (C) can be eliminated. Answer choice (A) is therefore correct.
You do not have the required permissions to view the files attached to this post.
 gmd114@miami.edu
  • Posts: 3
  • Joined: May 15, 2023
|
#102193
Not understanding
 Luke Haqq
PowerScore Staff
  • PowerScore Staff
  • Posts: 742
  • Joined: Apr 26, 2012
|
#102202
Hi gmd114!

I wasn't sure which specific part of this question you didn't understand, but I can try to unpack it in a way that is hopefully helpful to you.

To start, this is a could be true question. The correct answer choice could be true, whereas the four incorrect answer choices must be false. From that perspective, we can see off the bat whether there are answer choices that we know must be false based on the rules of the game.

This question is asking about which class could be scheduled for the afternoon on day one. The first rule is: "Day 2 is the only day for which oils can be scheduled." This allows us to eliminate answer choice (B).

The second rule is: "Neither sculpture nor watercolors can be scheduled for the afternoon." This allows us to eliminate answer choices (D) and (E).

Finally, we can also rule out (C). The last rule of the game is: "If pastels is scheduled for day 1 or day 2, then the lectures scheduled for the day immediately following pastels must be fresco and history, not necessarily in that order." Pastels can't be on day 1 in this question, because this rule tells us that fresco and history must be on day 2, but there's no room for that because lithography is on day 2.

Thus (B), (C), (D), and (E) cannot be true. The only answer that could be true is (A).

Get the most out of your LSAT Prep Plus subscription.

Analyze and track your performance with our Testing and Analytics Package.